Matematik

Komplekse tegnplan

28. marts 2017 af UchihaItachi - Niveau: Universitet/Videregående

Hej, hvordan kan jeg løse den opgave: 

Tallet w opfylder, at \left | w \right |=2*4+4 og arg(w)=\left ( \frac{1}{3}*4-\frac{3}{2} \right )*\pi

b) Indtegn w i den komplekse talplan. 

Er ikke helt med på hvordan det gøres.. 

Tak


Brugbart svar (1)

Svar #1
28. marts 2017 af mathon

komplekse talplan:

                                       w=12\cdot e^{i\cdot \left ( -\frac{\pi }{6} \right )}=12\cdot \cos\left ( -\frac{\pi }{6} \right )+12\cdot i\cdot \sin\left (- \frac{\pi }{6} \right )

                                       w=6\sqrt{3}-6i


Svar #2
28. marts 2017 af UchihaItachi

Det er jeg også kommet frem til, men besvare dette spørgsmål ikke mit næste spørgsmål c)?

Spørgsmål c er:

Bestem w på rektangulær form


Svar #3
28. marts 2017 af UchihaItachi

Derfor var jeg usikker på spørgsmål b og måske om det var noget man skulle tegne


Brugbart svar (1)

Svar #4
28. marts 2017 af mathon

Indtegnes
                         \left ( 12,\angle-30^\circ \right )=\left ( 12,\angle-\tfrac{\pi }{6} \right )


Svar #5
28. marts 2017 af UchihaItachi

Kan dette også gøres på maple? Hvis ja hvordan? 


Brugbart svar (0)

Svar #6
28. marts 2017 af mathon

maple regner i radianer.


Svar #7
28. marts 2017 af UchihaItachi

Ja okay, hvordan skriver jeg det på maple :)


Skriv et svar til: Komplekse tegnplan

Du skal være logget ind, for at skrive et svar til dette spørgsmål. Klik her for at logge ind.
Har du ikke en bruger på Studieportalen.dk? Klik her for at oprette en bruger.